Skip to main content

Preparation Manual

Print this page

Section 4: Sample Selected-Response Questions
Science of Teaching Reading (293)

Expand All Answers | Collapse All Answers

This section presents some sample exam questions for you to review as part of your preparation for the exam. To demonstrate how each competency may be assessed, sample questions are accompanied by the competency that they measure. While studying, you may wish to read the competency before and after you consider each sample question. Please note that the competency statements do not appear on the actual exam.

The sample questions are included to illustrate the formats and types of questions you will see on the exam; however, your performance on the sample questions should not be viewed as a predictor of your performance on the actual exam.

Selected-Response Questions with Rationales

Each sample exam question here includes the correct answer and a rationale for each answer option.

Domain I—Reading Pedagogy

Competency 001—(Foundations of the Science of Teaching Reading): Understand foundational concepts, principles, and best practices related to the science of teaching reading.

1. Which of the following practices by a prekindergarten teacher best reflects an assets-based approach to reading instruction?

  1. planning instruction in various areas of reading using continually adjusted flexible groupings according to each child's current assessed knowledge and skills
  2. allowing individual children to develop an interest in literacy at their own pace before introducing them to foundational reading concepts and skills
  3. ensuring that each child in an emergent-reading group has mastered the current reading concept or skill before moving the group on to the next lesson
  4. building opportunities in each lesson for children to use multiple modalities to deepen their understanding of new reading concepts and skills
Enter to expand or collapse answer.Answer expanded
Option A is correct because an assets-based approach to reading instruction focuses on what children know rather than on what they do not know. By continually adjusting flexible groupings according to current assessment data, the teacher acknowledges each child's growth in various areas of reading. The use of ongoing assessment also supports the teacher in identifying each child's assets in order to create student-centered instruction. Option B is incorrect because in an assets-based approach, the teacher does not hold children back until they demonstrate an interest in literacy. Option C is incorrect because the strategy described locks children into a particular reading group without consideration of individual children's growth. Option D is incorrect because the focus of this strategy is using multiple modalities for instruction rather than focusing on what the child brings to the learning context and building on those strengths.

2. A first-grade student has been identified as having dyslexia and has begun intervention. Which of the following approaches to instruction would be most effective to enhance the student's reading development?

  1. allowing the student to use colored overlays on all classroom texts to ameliorate the visual difficulties caused by dyslexia
  2. using reading materials with the student for instruction and guided practice that utilize specialized fonts designed for people with dyslexia
  3. arranging for the student to spend time each day on the classroom computer using a working-memory training program
  4. providing the student with systematic, explicit multimodal instruction in all the essential, evidence-based components of reading
Enter to expand or collapse answer.Answer expanded
Option D is correct because convergent research on dyslexia supports a language-based, multimodal approach to instruction that is systematic and explicit; addresses all five components of reading (phonemic awareness, phonics, fluency, vocabulary, and text comprehension); and integrates spelling and writing instruction with reading instruction. Language-based means that attention is given to all the major language systems (phonology, morphology, semantics, syntax, and discourse). Multimodal instruction involves using auditory, visual, and tactile-kinesthetic sensory systems along with articulatory-motor components to help students link spoken language to the printed language on the page. It also involves integrating listening, speaking, reading, and writing activities to reinforce new learning across language modalities. Systematic means that the teacher uses a planned sequence of evidence-based instruction with an emphasis on developing accuracy and automaticity in all skills. The sequence of instruction follows an optimal order for introducing new information and skills according to the increasing complexity and/or relative utility of linguistic units. Explicit means that language structures (e.g., phonemes, letter-sound relationships, syllable types) and skills are taught directly to students using modeling and/or demonstration, explanation, examples, teacher-guided practice, and independent practice leading to automaticity. Options A and B are incorrect because these strategies are based on the false premise that dyslexia is a visual condition. Convergent research has found these strategies to be ineffective in addressing the underlying linguistic difficulties involved in dyslexia. Option C is incorrect because it is based on the false premise that dyslexia is caused by a deficit in working memory. As with the strategies described in options A and B, convergent research has shown working-memory training programs to be ineffective in treating dyslexia.

For more detailed information on dyslexia and its treatment, see The Dyslexia Handbook: Procedures Concerning Dyslexia and Related Disorders (2018), Texas Education Agency. https://tea.texas.gov/sites/default/files/2018-Dyslexia-Handbook_Approved_Accomodated_12_11_2018.pdf opens in new window

Competency 002—(Foundations of Reading Assessment): Understand foundational concepts, principles, and best practices related to reading assessment.

3. A third-grade teacher frequently uses an online application at the end of a lesson that allows the teacher to post a small task or question for students on the classroom computer. For example, after a lesson on prefixes, the teacher posts three base words and asks students to change the meaning of each word by adding an appropriate prefix from the lesson. Throughout the day, students post their individual responses for the teacher to review. In this scenario, the teacher is using technology for which of the following assessment purposes?

  1. formative assessment
  2. diagnostic assessment
  3. summative assessment
  4. screening assessment
Enter to expand or collapse answer.Answer expanded
Option A is correct because the purpose of a formative assessment is to obtain information about what students have learned and are able to do following instruction. In the scenario, the teacher uses the online application as an exit ticket to check individual students' learning with respect to given reading lessons (e.g., instruction in prefixes). The teacher can use the data collected to inform and adjust subsequent instruction. Option B is incorrect because the purpose of a diagnostic assessment is to assess a specific area or areas of reading in depth to draw conclusions about the cause(s) of a student's reading difficulties. Option C is incorrect because a summative assessment is typically administered at the end of an instructional unit or period of time (end of semester, end of school year) to measure students' progress in reading with respect to specific learning standards. Option D is incorrect because a screening assessment is typically administered in order to measure discrete skills that could identify a student or students as being at-risk for developing reading difficulties.

Domain II—Reading Development: Foundational Skills

Competency 004—(Phonological and Phonemic Awareness): Understand concepts, principles, and best practices related to the development of phonological and phonemic awareness, and demonstrate knowledge of developmentally appropriate, research- and evidence-based assessment and instructional practices to promote all students' development of grade-level phonological and phonemic awareness skills.

4. At the beginning of the school year, a first-grade teacher conducts a brief screening assessment in which the teacher asks small groups of students to spell four CVC words and one word with a consonant blend (e.g., bag, hen, sit, mop, slug). In addition to providing the teacher with information about students' knowledge of letter-sound correspondences, this type of assessment would also provide information about students' development in which of the following other areas related to emergent reading?

  1. vocabulary knowledge
  2. phonological awareness
  3. phonemic awareness
  4. listening comprehension
Enter to expand or collapse answer.Answer expanded
Option C is correct because in this assessment students are representing phonemes (sounds) with graphemes (letters). The number of graphemes a student writes and the sequence of the graphemes provides insight into the students' phonemic awareness, specifically their phonemic segmentation skills. If a student spells a CVC word with one letter that represents the beginning sound, it indicates that the student most likely perceives only the initial or most salient sound in a word. If the student spells a CVC word with both the beginning and ending consonants, this suggests that the student can perceive the initial and final phoneme of a word. If the student spells a target word with a beginning and an ending consonant along with a vowel in the middle, even if it is the wrong vowel, this indicates the ability to perceive three sounds. Likewise, if a student spells a target word that contains a blend (e.g., slug) using only three letters (e.g., sug), this suggests that the student likely can perceive words with up to three phonemes but not four phonemes. Thus, by analyzing students' spellings in this brief screening assessment, a teacher can draw some conclusions about a student's ability to perceive phonemes in words (i.e., phonemic awareness). This information can help inform instruction in phonemic awareness to support students' spelling and decoding development. Option A is incorrect because the assessment is not measuring students' knowledge of word meaning. Option B is incorrect because phonological awareness skills include the ability to perceive and manipulate many types of linguistic units (e.g., word, rhyme, syllable, onset/rime), while the narrower skill of phonemic awareness specifically relates to perceiving and manipulating individual phonemes in words. Option D is incorrect because spelling at this stage of literacy development relates exclusively to the phonological structure of language—the rendering of phonemes as graphemes. Whereas, listening comprehension relates to the meaning of language, typically at the sentence or discourse levels (discourse being a text or utterance longer than a single sentence [e.g., a paragraph, a story, an article, a chapter, a conversation, a lecture]).

5. A prekindergarten teacher is preparing an introductory lesson focused on isolating/identifying the initial sound in spoken words for a small group of children whose informal assessments indicate that they are ready to learn this skill. The group includes an English learner. Which of the following instructional supports would best promote the English learner's success in achieving the instructional goal of this lesson?

  1. selecting stimulus words for the lesson that have sounds common to both English and the English learner's home language
  2. developing an alternative lesson for the English learner that focuses on skills that fall earlier along the phonological awareness continuum
  3. conducting tasks with the English learner that require the child to segment and blend the sounds of simple spoken English words
  4. encouraging the English learner's family to engage in wordplay activities with their child such as reciting home-language nursery rhymes
Enter to expand or collapse answer.Answer expanded
Option A is correct because when focusing on the skill of isolating/identifying phonemes (sounds) in spoken words, English learners benefit from stimulus words that contain familiar sounds. Research has shown that babies as young as ten months stop attending to and producing speech sounds that do not occur in their home language(s). By the time children are school age, they may require explicit instruction to be able to perceive phonemes that are not present in the home language(s). By selecting words for instruction that have initial sounds that the English learner can perceive and produce already, the teacher ensures that the English learner is on the same footing as the English-speaking peers in the group; that is, the child has the same potential to benefit from instruction. Option B is incorrect because the scenario states that assessments indicate that the English learner is ready to progress to instruction in phonemic awareness. Continuing to present the child with tasks that occur earlier in the continuum of phonological awareness skills would mean preventing the child from making progress in emergent literacy development. Option C is incorrect because segmenting and blending all the sounds in a spoken word are more challenging skills than the focus of this lesson; that is, these skills occur later in the developmental continuum of phonological and phonemic awareness. According to evidence-based best practices in the development of phonemic awareness, instruction should be sequenced according to the increasing complexity of linguistic units/skills. Option D is incorrect because, like option B, it does not advance the child along the phonological and phonemic awareness continuum. Rhyming is a phonological awareness skill, whereas the child in the scenario is ready to progress to learning phonemic awareness skills.

6. As part of an informal assessment of students' phonemic awareness skills, a kindergarten teacher meets with individual students and says, "We're going to play a word game. I'm going to say a word that you know. When you hear it, I want you to say each sound in the word in the right order. For example, if I say fan, you should say ." The teacher then helps the student practice the procedure using the practice words in, sat, and top. After meeting with each student, the teacher reviews students' performance and notices that several students performed similarly on the assessment. A representative sample of their assessment results is shown below.

Target Word Student Response
at short a sound, segment marker, t sound
men m sound, segment marker, en
line l sound, segment marker, ine
hot h sound, segment marker, ot
gum hard g sound, segment marker, um

Given the information provided, which of the following student activities would be most appropriate for the teacher to include when planning differentiated instruction to promote the students' growth in phonemic awareness?

  1. identifying and matching the initial, medial, and final sounds of words represented by pictures
  2. practicing reading simple words that belong to common word families
  3. identifying and matching pictures that represent words with the same onset or rime
  4. practicing reading a variety of simple decodable texts
Enter to expand or collapse answer.Answer expanded
Option A is correct because the task described in this scenario–asking students to say each sound in a word in the correct order—is an example of a phoneme-segmentation task. An analysis of the students' responses to the target words suggest that the students can segment words into onset and rime (e.g., g/um) but not into individual phonemes. The teacher needs to promote the students' ability to perceive individual sounds in words at a skill level that is in between onset/rime segmentation and phoneme segmentation. Having the students practice identifying the initial, medial, and final sounds of words represented by pictures builds on what they can do to move them to the next skill level along the phonological and phonemic awareness continuum. Option B is incorrect because reading word families should not be introduced until after students have consolidated their basic phonics skills and are ready to learn to read certain sets of words (i.e., words that follow syllable patterns already taught) with automaticity. In addition, working with word families would focus the students on the same onset/rime skill that they have already learned and would not help them learn to attend to each of the individual sounds in a word. Option C is incorrect because the assessment data indicate that the students can already segment words into onset and rime. Option D is incorrect because the students in the scenario are not yet attending to all the phonemes in a word. By skipping over critical phonemic awareness skills, the teacher would leave the students without appropriate foundational skills to support their progress in word-reading and spelling development. Convergent research has shown that gaps in phonemic awareness skills frequently cause students to experience difficulties in decoding and/or spelling in later grade levels. Note that after the teacher in this scenario begins working with the students to promote their ability to identify the initial, final, and medial phonemes in spoken CVC words, the students are likely to benefit from the teacher reinforcing the instruction by supporting them in sounding out relevant words in simple decodable texts (Ehri, 2014). However, in the given scenario, the students are not likely ready to benefit from reading decodable texts independently.

Competency 005—(Print Concepts and Alphabet Knowledge): Understand concepts, principles, and best practices related to the development of print concepts and alphabet knowledge, including understanding of the alphabetic principle, and demonstrate knowledge of developmentally appropriate, research- and evidence-based assessment and instructional practices to promote all students' development of grade-level print concepts and alphabet knowledge and their understanding of the alphabetic principle.

7. A kindergarten teacher is planning instruction for a small group of students who have mastered the letter-sound relationships for the consonants m, s, t, and p and for the short-vowel sound of the letter a. The students also consistently spell words using both initial and final consonant sounds in their daily writing. Given this information, which of the following instructional activities would be most appropriate for the teacher to use with these students to promote their transition to the next step along the continuum of development of knowledge and skills related to the alphabetic principle?

  1. exploring the letter-sounds for letter names that do not contain the sound that the letter represents (e.g., h, w)
  2. introducing the students to early decodable texts featuring known letter-sound relationships and modeling how to sound out the words
  3. consolidating the students' knowledge of letters that contain the letter's sound at the beginning of the letter's name (e.g., b, d, j, k, q)
  4. practicing writing each letter of the alphabet in response to a spoken prompt of the letter's sound to build motor memory for letter-sound relationships
Enter to expand or collapse answer.Answer expanded
Option B is correct because the kindergarten students described in the scenario are able to spell words with the letter-sound relationships they know. This suggests that they understand the alphabetic principle and can perceive at least two phonemes in words. Because of the reciprocity between encoding and decoding, the students are ready to begin reading early decodable texts that feature the letter-sound relationships they know. They will benefit from teacher support (e.g., modeling how to sound out words) to reinforce their understanding of the decoding process. Typically, early decodable texts are designed so that students have the opportunity to learn and apply one short-vowel sound at a time, with additional short vowels added into successive texts systematically. Options A and C are incorrect because they focus instruction in isolated letter-sounds. These strategies would not move students toward decoding of words nor take advantage of the progress the students have made related to understanding the alphabetic principle. Option D is incorrect because, although it reinforces fluency in recognizing and forming letters, it would not move the students to the next step of applying their knowledge of letter-sound relationships to sound out words.

8. A prekindergarten teacher is planning instruction in letter-sound relationships for a group of beginning-level English learners who have begun identifying and naming the letters of the alphabet. Which of the following strategies would likely be most effective to apply with this group of children?

  1. using oral translations of tongue twisters from the children's home languages to draw their attention to letter-sounds that are common between the two languages
  2. ensuring that the children can distinguish between illustrations and print and identify the parts of a printed sentence as prerequisites to learning letter-sounds
  3. employing articulatory feedback to help the children discover English letter-sounds that are not in their home language and learn how to pronounce them
  4. showing the children how to use a variety of simple mnemonic devices to help them memorize unfamiliar English letter-sounds
Enter to expand or collapse answer.Answer expanded
Option C is correct because English learners learning letter-sound relationships that are not in their home language benefit from articulatory feedback to discover how to pronounce the sounds. Articulatory feedback includes feeling the position of the tongue and/or lips when making the sound and feeling for the airstream or puff of air. Articulatory feedback can also include determining if the sound vibrates (is vocalized) or not when it is produced (e.g., compare /f/ and /v/) or passes through the nose (e.g., /n/, /m/, /ng/). Convergent research recommends using articulatory feedback as a multimodal instructional support for teaching all students letter-sounds, but it is especially critical for English learners for whom certain English sounds may be unfamiliar. Option A is incorrect because the sounds of words in tongue twisters may not transfer between languages. Option B is incorrect because distinguishing between illustrations and print or identifying the parts of a printed sentence are not prerequisite skills for learning letter-sound relationships. Option D is incorrect because it does not address how to produce unfamiliar English letter-sounds. Children will have difficulty remembering sounds that they cannot perceive and/or produce.


Competency 006—(Phonics and Other Word Identification Skills): Understand concepts, principles, and best practices related to the development of phonics and other word identification skills, including related spelling skills, and demonstrate knowledge of developmentally appropriate, research- and evidence-based assessment and instructional practices to promote all students' development of grade-level phonics and other word identification skills and related spelling skills.

9. A second-grade teacher frequently uses the strategy of phoneme-grapheme mapping as part of phonics instruction. The teacher selects target words from a phonics lesson and creates sound boxes corresponding to the words. The teacher then helps students write the target words in the sound boxes, making sure that students map each sound of a word to a single box. Examples of sound boxes from two different phonics lessons are shown below.

Sample sound boxes for one group of students:

Target word: chain. The first box contains the letters c h. The second box contains the letters a i. The third box contains the letter n.

Target word: cheap. The first box contains the letters c h. The second box contains the letters e a. The third box contains the letter p.

Sample sound boxes for a second group of students:

Target word: bent. The first box contains the letter b. The second box contains the letter e. The third box contains the letter n. The fourth box contains the letter t.

Target word: fist. The first box contains the letter f. The second box contains the letter i. The third box contains the letter s. The fourth box contains the letter t.

Using this activity in the context of phonics lessons best demonstrates the teacher's understanding of which of the following key concepts related to beginning reading instruction?

  1. the importance of sequencing phonics instruction according to the increasing complexity of linguistic units
  2. the relationship between accurate, automatic decoding and the development of reading fluency and comprehension
  3. the importance of utilizing the reciprocity between decoding and encoding to reinforce phonics instruction
  4. the relationship between providing students with frequent practice with decodable text and the development of automaticity
Enter to expand or collapse answer.Answer expanded
Option C is correct because phoneme-grapheme mapping is a visual activity that allows students to map graphemes onto the individual phonemes or sounds they represent in a word. A grapheme is a written symbol that represents a sound. A grapheme can be a single letter or a sequence of letters (e.g., ea, -tch). In a phoneme-grapheme map, each box is a sound box and only one sound can go into each box. So, phoneme-grapheme mapping provides students with a visual aid to reinforce phonics elements, such as consonant digraphs (e.g., ch) and vowel teams (e.g., ai, ea), which each make one sound and therefore appear in one sound box, and consonant blends (e.g., -nt, ‑st), which make two sounds and therefore must be mapped to two sound boxes. By engaging in phoneme-grapheme mapping, students both hear and see the relationship between letters, letter combinations, and the sounds they represent. Option A is incorrect because the scenario does not mention the instructional sequence of the phonics skills being taught to the two groups of students. One can discern that the students in the first group have already learned how to map closed-syllable words and have moved on to mapping vowel-team-syllable words, but the scenario does not focus on this aspect of instruction. Option B is incorrect because the scenario does not focus on the relationship between accurate, automatic decoding and the development of reading fluency and comprehension. Option D is incorrect because the scenario does not mention the use of decodable text or its relationship to the development of automaticity.

10. A kindergarten teacher reads a decodable text about cats with a small group of students and then incorporates the content of the text into an interactive writing lesson. First, the teacher has students orally generate several sentences that relate to the actions of the cat in the story. The teacher then says, "Those are great sentences. Help me write them on the chart paper." For each decodable word in a sentence, the teacher pauses to prompt the students to listen to the sounds of the word and use their knowledge of the letter-sound correspondences that they practiced in the decodable text to identify which letter the teacher should write next. This scenario best demonstrates the teacher's awareness of which of the following concepts related to students' development of beginning reading skills?

  1. the role of basic print concepts in writing development
  2. the importance of fostering students' motivation to read and write
  3. the role of accurate, automatic decoding in fluent writing
  4. the importance of applying newly taught phonics elements to writing
Enter to expand or collapse answer.Answer expanded
Option D is correct because the scenario describes an interactive writing lesson designed to prompt students to apply what they know about phonics to writing words in a meaningful context. The teacher models the thinking process of going from an idea to print on the page. The teacher knows the phonics elements students have learned during instruction, so as the teacher is writing each sentence, when the teacher comes to words that consist of known phonics elements, the teacher asks the students to apply those letter-sound relationships to spell the words. This type of interactive writing routine, in which the students "share the pen" with the teacher, provides students with opportunities to apply newly taught phonics elements to meaningful writing by using the phonics skills they just used to decode the words in the story to construct meaningful sentences about the story. Convergent research has shown that students benefit from practicing spelling the same words that they are learning to read. As an added bonus of this instructional strategy, research also strongly suggests that writing about what they have read strengthens students' reading comprehension. Options A and C are incorrect because the focus of the instruction described in the scenario is not on developing students' basic print concepts or promoting their automatic decoding. Option B is incorrect because, while promoting students' motivation to read and write may be a secondary outcome of the lesson, the primary focus of the lesson is clearly on reinforcing newly taught phonics elements through meaningful writing.

Competency 007—(Syllabication and Morphemic Analysis Skills): Understand concepts, principles, and best practices related to the development of syllabication and morphemic analysis skills, including related spelling skills, and demonstrate knowledge of developmentally appropriate, research- and evidence-based assessment and instructional practices to promote all students' development of grade-level syllabication and morphemic analysis skills and related spelling skills.

11. A first-grade teacher would like to incorporate instruction in morphemes for students who have mastered reading and spelling closed- and open-syllable words. Which of the following skills is best aligned with both the teacher's goal and the continuum of word-reading skills described in the first-grade Texas Essential Knowledge and Skills (TEKS) for English Language Arts and Reading (ELAR)?

  1. decoding and identifying the meaning of words with the inflectional endings -s, -es, and -ed
  2. decoding and spelling words with vowel teams and r-controlled syllables
  3. decoding and identifying the meaning of words with the affixes un-, re-, -er, and -est
  4. decoding and spelling words with initial and final consonant blends and digraphs
Enter to expand or collapse answer.Answer expanded
Option A is correct because in first grade, morpheme instruction should focus on decoding and identifying the meaning of words with the inflectional endings -s, -es, and -ed, according to the Texas Essential Knowledge and Skills (TEKS) for English Language Arts and Reading (ELAR). Evidence-based morphological research suggests that kindergarten and first-grade students demonstrate emerging morphological awareness and that this awareness increases with explicit instruction. The teacher should teach the students how to apply inflectional endings to words that follow phonics patterns that the students already know how to read. The teacher can easily explain or demonstrate the meaning of these inflectional endings (i.e., plural markers and past tense marker, respectively). Options B and D are incorrect because they refer to the development of phonics or orthographic knowledge rather than knowledge of morphemes, which are units of meaning. Option C is incorrect because it describes derivational morphemes that are typically taught after first grade in the continuum of word-reading skills described in the TEKS for ELAR.

Competency 008—(Reading Fluency): Understand concepts, principles, and best practices related to the development of reading fluency, and demonstrate knowledge of developmentally appropriate, research- and evidence-based assessment and instructional practices to promote all students' development of grade-level reading fluency.

12. A second-grade student scores well above the 50th percentile benchmark for fluency on the midyear benchmark assessment. However, the teacher notes that the student reads the text word-by-word in a choppy, disjointed manner and has difficulty answering comprehension questions afterward. Which of the following strategies would be most important for the teacher to include in an intervention designed to address the student's assessed needs?

  1. engaging in a systematic review of phonics elements that should be mastered by the middle of second grade
  2. engaging in silent wide-reading of books written at the student's independent reading level
  3. engaging in oral reading following teacher modeling using texts that are phrase-cued to approximate speech
  4. engaging in buddy reading and choral reading with another student who reads at the same level
Enter to expand or collapse answer.Answer expanded
Option C is correct because the description of the student's reading suggests difficulty with prosody, one of the key indicators of reading fluency. Students who read "word-by-word in a choppy, disjointed manner" will likely have difficulty understanding what they read. One feature of prosody is reading with proper phrasing, to capture the meaning of the text and/or the author's intent. To scaffold prosodic reading, the teacher can draw swoops under phrases in a section of a text to represent speech-like phrasing. The teacher can then model how to read the phrase-cued text with appropriate intonation, pitch, and expression that conveys the meaning expressed by the words. Options A and B are incorrect because these two strategies are designed to promote students' decoding accuracy and automaticity, respectively. Since the scenario states that the student "scores well above the 50th percentile benchmark for fluency on the midyear benchmark assessment," this suggests that the student's difficulty with prosody is not based on deficits in decoding. Option D is incorrect because it does not provide the student with explicit instruction designed to improve the student's prosody.

13. Several students in a first-grade class have progressed from the partial-alphabetic phase of word-reading development to the full-alphabetic phase. Which of the following instructional activities would be most appropriate for promoting these students' word-reading accuracy and automaticity?

  1. having the students practice reading simple closed-syllable words in isolation and in decodable texts
  2. modeling the use of contextual strategies to read unfamiliar words
  3. increasing the amount of time the students spend practicing reading irregular high-frequency words
  4. supporting the students in reading predictable texts in a variety of genres
Enter to expand or collapse answer.Answer expanded
Option A is correct because students in the full-alphabetic phase of word-reading development need multiple exposures to explicitly taught syllable types to consolidate their learning and develop automaticity, an essential component of reading fluency. Students in the earlier, partial-alphabetic phase of word reading read words by forming partial connections between the more salient sounds in a word and the sounds' graphemes, or letter representations. Students at this stage primarily rely on predicting words from their initial letters and context. Students advance to the full-alphabetic phase when they can form complete connections between graphemes and phonemes. Students at this stage sound out closed-syllable words into individual sounds by using their phonemic segmentation and blending skills and their growing phonics knowledge. At first, students will sound out and blend each letter-sound of a word. However, when they are given opportunities to sound out words multiple times, they can accumulate words they have decoded and begin to recognize them automatically. Having the students practice reading simple closed-syllable words in isolation and in decodable text supports this process and allows students to recognize an increasing number of words automatically by sight, while orthographic knowledge continues to accumulate. According to L.C. Ehri's research (2014), once a word has been stabilized in memory through repeated exposures, the mere sight of the word's spelling immediately activates its pronunciation and meaning (automaticity). Options B and D are incorrect because modeling contextual strategies to read unfamiliar words or using predictable texts misdirects students from the essential practice they need decoding explicitly taught phonics patterns (syllable types) again and again to develop automaticity. Option C is incorrect because memorizing irregular high-frequency words does not help students become more automatic with regular closed-syllable words (one of the essential foundational building blocks of orthographic knowledge in English).

14. Before assigning students a new science or social studies text, a third-grade teacher introduces important Tier Three terms from the text. As part of the introduction, the teacher leads students in applying morphemic analysis skills to the words and also discusses new concepts related to the words. The teacher's actions best reflect an understanding of which of the following factors that can disrupt reading fluency and affect comprehension?

  1. lack of automaticity in decoding grade-level words
  2. unfamiliarity with a text's content
  3. unfamiliarity with complex grammatical structures
  4. limited phonics knowledge or skills
Enter to expand or collapse answer.Answer expanded
Option B is correct because the teacher's actions in leading students in applying morphemic analysis skills to key Tier Three words in the text and discussing new concepts related to the words demonstrate the teacher's awareness that vocabulary knowledge and concept development are closely interrelated—discipline-specific vocabulary development involves concept learning and concept learning supports academic vocabulary development. Convergent research has shown that reading fluency and reading comprehension break down if the reader is unfamiliar with too many words in a text. Tier Three terms are vocabulary words that are specific to a discipline or subject matter. Often informational texts have a great density of Tier Three terms, and students would benefit from strategies to analyze the words' structure into morphemes. Morphemic analysis allows students to begin to recognize common Greek and Latin roots and affixes that make up many science and social studies vocabulary words (e.g., demographic, democracy, democratic). Meanwhile, concept learning reinforces their academic-vocabulary development. Options A, C, and D are incorrect because the teacher's instruction in the scenario is clearly designed to promote students' development of background knowledge related to the text, including relevant vocabulary and concepts. The instruction is not focused on promoting decoding and automaticity, teaching complex grammatical structures, or improving phonics knowledge or skills.

Domain III—Reading Development: Comprehension

Competency 009—(Vocabulary Development): Understand concepts, principles, and best practices related to vocabulary development, and demonstrate knowledge of developmentally appropriate, research- and evidence-based assessment and instructional practices to promote all students' development of grade-level vocabulary knowledge and skills.

15. Which of the following sets of words would be most appropriate to categorize as Tier Two words?

  1. fossil, air, soil
  2. clock, book, floor
  3. arrange, observe, predict
  4. custom, shelter, community
Enter to expand or collapse answer.Answer expanded
Option C is correct because Tier Two words are words that represent more sophisticated or academic versions of everyday vocabulary. For example, arrange is a more academic word (verb) for place or put in order, observe is a more academic word for look, and predict is a more academic word for guess. Because Tier Two words are used less frequently in social language, students often find them challenging because they primarily encounter them in print. However, many Tier Two words have high utility in academic contexts. In contrast to Tier Three or discipline-specific words, Tier Two words are considered "general academic" words because they can be used across disciplines. Options A and D are incorrect because these groups of words are discipline-specific terms, and would therefore be categorized as Tier Three words. Option B is incorrect because clock, book, and floor are Tier One words. They represent everyday vocabulary commonly used in social language. However, it is important to note that while most Tier One words are generally very familiar to students whose home language is English, some Tier One words may be unfamiliar to students in general or to English learners in particular. Teachers should adjust vocabulary instruction, as needed.

Competency 010—(Comprehension Development): Understand concepts, principles, and best practices related to the development of reading comprehension, and demonstrate knowledge of developmentally appropriate, research- and evidence-based assessment and instructional practices to promote all students' development of reading comprehension strategies in order to gain, clarify, and deepen understanding of appropriately complex texts.

16. A third-grade English learner has grade-level decoding skills and scores around the grade-level benchmark for words correct per minute on oral reading fluency measures. However, the student's text comprehension is mixed. The student comprehends some literary and informational texts with ease, yet struggles with others. Given this evidence, when the student is having difficulty with a text, the teacher's best initial response should be to:

  1. provide the student with a list of probing questions to answer after the student finishes reading the text.
  2. talk with the student to informally assess the extent of the student's background knowledge about the text's topic or setting.
  3. advise the student to read the text more slowly and to focus on comprehension rather than decoding accuracy.
  4. encourage the student to develop a written summary of the text's key events or central ideas as the student is reading the text.
Enter to expand or collapse answer.Answer expanded
Option B is correct because the scenario demonstrates the important consideration of background knowledge in students' reading comprehension of both informational and literary texts. The type and amount of prior knowledge students have accumulated related to a particular topic or setting can vary considerably. English learners often experience the additional challenge of having had limited or no exposure to key vocabulary related to the topic or setting of a given text. The fact that the English learner's reading comprehension in the scenario varies from text to text suggests that the student's difficulty is most likely caused by gaps in background knowledge and related vocabulary within the given text. Convergent research on reading comprehension suggests that background knowledge, which includes related vocabulary knowledge (because vocabulary knowledge and concept development are often interdependent), are strong predictors of reading comprehension. These factors can also indirectly influence whether a student can successfully apply comprehension-repair strategies when meaning breaks down. A teacher can informally assess the extent of a student's background knowledge about a text's topic or story setting as a way to determine the amount of instructional support the student may need to gain, clarify, and deepen their understanding of the text. Option A is incorrect because this strategy is dependent on the student being able to make enough sense of the text to answer probing questions about it. Option C is incorrect because the assessment data given in the scenario suggests that the student is not experiencing particular difficulty with decoding accuracy. Option D is incorrect because summarizing the key events or central ideas in a text is dependent on having the background knowledge and related vocabulary necessary to make sense of the text.

17. Students in a second-grade class will be reading a complex informational text about ants as part of a science unit focused on comparing the ways living organisms depend on one another. Prior to the reading, the teacher plans to show students a video depicting activities in ant colonies and to share a picture book about ants. The teacher's actions best demonstrate understanding of which of the following factors affecting reading comprehension?

  1. ability to read fluently with prosody
  2. ability to monitor for understanding
  3. the role of vocabulary knowledge
  4. the role of background knowledge
Enter to expand or collapse answer.Answer expanded
Option D is correct because the teacher in the scenario is focusing on systematically building the students' background knowledge related to the theme of the unit—"how living organisms depend on one another." By showing a video depicting activities in ant colonies and reading aloud and discussing a picture book about ants, the teacher is building students' background knowledge related to the unit's theme, thereby providing them with the content knowledge they will need to access the more complex text about ants they will be reading in the science unit. Moreover, by providing students with both visual and oral language experiences with the new content through the video and picture book activities, the teacher is more likely to ensure that the content will be accessible to all students in the class. Option A is incorrect because prosody is a key indicator of reading fluency, and the activities described in the scenario focus on viewing and oral language activities to support concept development. While enhanced knowledge of the topic of the text is likely to help support students' prosody when reading the complex text, the teacher does not specifically model prosodic reading of the text or provide explicit instruction in specific aspects of prosody in the given lesson. Option B is incorrect because the teacher in the scenario does not model or explicitly teach behaviors related to self-monitoring for comprehension during reading. Option C is incorrect because the scenario does not suggest the teacher explicitly teaches vocabulary during this lesson, although the students' exposure to relevant vocabulary from the video and picture book will likely play a part in building their background knowledge. Moreover, the scenario describes the video as "depicting activities in ant colonies." Clearly, the teacher is focusing on systematically building the students' background knowledge related to the theme of the unit.

Competency 011—(Comprehension of Literary Texts): Understand concepts, principles, and best practices related to the comprehension of and critical thinking about literary texts, and demonstrate knowledge of developmentally appropriate, research- and evidence-based assessment and instructional practices to promote all students' development of grade-level comprehension and analysis skills for literary texts.

18. A third-grade class includes several English learners who represent a variety of home languages and English language proficiency levels. Some of the students are beginning readers in English. The teacher collects folktales from several countries to use for whole-class read-alouds and during small-group reading instruction. Which of the following statements accurately describes a feature of folktales and why that feature makes the genre especially well-suited for students in a multilingual, multicultural classroom?

  1. Folktales usually provide a resource for integrating a variety of hands-on arts and crafts projects into a reading lesson.
  2. Folktale plots tend to be compelling because they explore the human condition in depth.
  3. Folktale themes tend to be universal, so students are likely to have the necessary schema to comprehend them.
  4. Folktales usually include archaic vocabulary, which students are likely to find amusing.
Enter to expand or collapse answer.Answer expanded
Option C is correct because folktales convey universal themes common to most cultures, such as the virtues of compassion, generosity, and humility succeeding over the vices of greed, selfishness, and excessive pride or boastfulness. In addition, different versions of some folktales exist across cultures (e.g., Cinderella stories). Thus, students in a multilingual, multicultural classroom are likely to have the necessary schema, or background knowledge, to comprehend the stories because of the universality of these themes. Option A is incorrect because, while an arts and crafts project might enrich students' educational experience, it will not directly benefit students' understanding of the central theme of a folktale. Option B is incorrect because folktales are typically short, didactic narratives intended to teach a basic moral or explain a particular construct or phenomenon. They do not explore the human condition in depth, as a novel might do, for example. Option D is incorrect because students are likely to find archaic vocabulary more challenging than amusing. This is not a feature that would be especially well-suited to a multilingual, multicultural setting.

Competency 012—(Comprehension of Informational Texts): Understand concepts, principles, and best practices related to the comprehension of and critical thinking about informational texts, and demonstrate knowledge of developmentally appropriate, research- and evidence-based assessment and instructional practices to promote all students' development of grade-level comprehension and analysis skills for informational texts.

19. A kindergarten teacher frequently conducts think-alouds when reading aloud to the class. The following is a transcript of the teacher reading aloud an informational text called "Making a Bird Feeder." The teacher's think-aloud process is seen below in italics.

start italics Text end italics:  Making a Bird Feeder.

start italics Materials end italics:  one two-foot piece of string, one pinecone, honey, seeds, two bowls, one coat hanger

start italics Instructions end italics

Step 1:  Get the materials together.
Step 2:  Tie the string to the top and bottom of the pinecone.(Pausing) Hmm, I wonder why the instructions say to tie the string at each end? I'm going to keep reading and see.
Step 3:  Pour honey in one bowl and seeds into the other bowl.
Step 4:  Hold the string at each end, dip the pinecone in the honey, and then dip it in the seeds.(Pausing) Oh! Now I know why the instructions say to tie the string to each end! If I didn't, I think I would get honey all over my hands, and they would become sticky.
Step 5:  Tie each pinecone birdfeeder to a coat hanger until you are ready to hang them outside to attract bird friends.

Which of the following text analysis skills does the teacher model during this think-aloud?

  1. drawing conclusions about information in a text
  2. identifying a text's central idea and supporting evidence
  3. making predictions about information in a text
  4. locating evidence in a text to support an argument
Enter to expand or collapse answer.Answer expanded
Option A is correct because in the italicized text (the think-aloud) the kindergarten teacher models how to pose questions during reading and use background knowledge to draw conclusions about information in the text. The teacher uses information in the text and background knowledge about the stickiness of honey to draw a conclusion about why Step 2, tying the string to both ends of the pinecone, happens before the other steps. Options B, C, and D are incorrect because the teacher does not identify the text's central idea, make a prediction, or look for evidence to support an argument during the think-aloud.

Clustered Questions

Use the information below to answer the two questions that follow.

A kindergarten teacher reads aloud the big book The Little Yellow Chicken's House by Joy Cowley to a small group of students. In the story, the main character has to make a decision as to whether or not his friends should come into the warm cozy house he built to get out of the rain, since they refused his requests for help during construction of the house. The teacher has the students orally rehearse their opinion as to whether or not the unhelpful characters should be allowed into the house. The teacher asks the students to include in their responses the reason for their opinion. As each student tells an opinion, the teacher holds up a sign that says "because" to prompt students to add to their responses.

Student:  I think the little yellow chicken should not let his friends in.

Teacher:  (Holding up "because" sign)

Student:  ... because his friends were lazy, and the little yellow chicken's friends got soaked, and the little yellow chicken should ignore his friends, and he should not invite his friends in.

Teacher:  That's great that you explained why he shouldn't let them in the house. Can you tell me your opinion again and use "because" to explain why?

Student:  I think the little yellow chicken should not let his friends get in because they were lazy and didn't help him build the house.

Teacher:  (Holding up a sign that says "so") Now use the word so, and tell us what happened when the little yellow chicken ignored his friends.

Student:  It was raining, so his friends got soaked.

Competency 003—(Oral Language Foundations of Reading Development): Understand foundational concepts, principles, and best practices related to the development of oral language, including second-language acquisition, and demonstrate knowledge of developmentally appropriate, research- and evidence-based assessment and instructional practices to promote all students' development of grade-level oral language skills.

20. In the dialogue, the teacher's practice of prompting students with connecting words benefits their oral language development primarily by:

  1. reinforcing for students the importance of supporting literary analysis with relevant evidence from the text.
  2. modeling for students various grammatically correct sentences.
  3. promoting students' use of sentences and grammatical structures of increasing complexity.
  4. encouraging students to use Tier Two and Tier Three vocabulary words.
Enter to expand or collapse answer.Answer expanded
Option C is correct because, although younger students may have a reason for their answer, developmentally they may not be used to making complex grammatical statements to express cause and effect. When the teacher holds up the word because, the teacher scaffolds the student's response, prompting the student to give a reason without the teacher asking a direct question. Once the student offers the cause or the reason for the stated opinion, the teacher prompts the student into creating a more complex sentence using an appropriate conjunction or connecting word (e.g., because, so). The teacher in effect supplies the conjunction the student needs to construct a more complex sentence. Option A is incorrect because the students have already analyzed the literary text and identified evidence in the text to support an opinion. The teacher is trying to help them better articulate their ideas by using more complex language that clarifies causal relationships between story events. Option B is incorrect because the teacher did not actually model a grammatically complex sentence. Instead, the teacher prompted the student to produce a more complex grammatical structure by providing the conjunction as a scaffold. Option D is incorrect because the focus of the lesson was on helping students use common conjunctions to express their ideas more fully by constructing more complex sentences. The teacher did not focus on helping students use Tier Two or Tier Three words.

21. The teacher brings in a variety of building materials mentioned in the story The Little Yellow Chicken's House (e.g., straw, wood, stones) for the students to describe, sort, and categorize. The teacher also asks the students to describe how the character used the materials in his house. The teacher's actions best demonstrate attention to which of the following instructional goals?

  1. developing metacognitive skills
  2. developing oral language expression
  3. developing listening comprehension
  4. developing metalinguistic awareness
Enter to expand or collapse answer.Answer expanded
Option B is correct because the teacher uses the various attributes of the physical materials (e.g., straw, wood, stones) and how they were used by the character in the story to provide the students with opportunities to practice key language functions, such as describing and categorizing, in a meaningful context. In addition to enhancing young students' oral language development, this approach also supports their emerging text analysis skills by providing them with purposeful opportunities to continue engaging with and discussing the text. Option A is incorrect because developing metacognitive skills refers to teaching students to think about their thinking and using metacognitive strategies to improve comprehension. This was not the goal of the activity. Option C is incorrect because the activity primarily focuses on developing the students' expressive language skills rather than on their receptive language skills. Option D is incorrect because metalinguistic awareness refers to consciously thinking about or reflecting on the structure of language rather than on the content or message being conveyed. Whereas, the focus of the teacher's actions in this lesson was to use concrete objects to promote students' expressive language development and ability to talk about a literary text with greater detail and precision.

Use the information below to answer the two questions that follow.

A prekindergarten teacher frequently creates learning centers related to texts that the teacher has read-aloud. The teacher also uses read-alouds for teaching related academic vocabulary. For example, the teacher wants to introduce children to a variety of foundational STEM (i.e., science, technology, engineering, and mathematics) words (e.g., experiment, predict, measure, observe, cause, effect, compare, results), so the teacher reads aloud Lola Plants a Garden by Anna McQuinn.

After reading and discussing the book with the children, the teacher works with them to plant a classroom window garden using paper cups. The teacher has the children conduct simple experiments with the garden, such as watering some plants more than others to observe and compare the results. The teacher also extends the activity by sending home seeds and simple directions in students' home languages to support families in conducting and discussing simple window-garden experiments at home with their child.

Competency 009—(Vocabulary Development): Understand concepts, principles, and best practices related to vocabulary development, and demonstrate knowledge of developmentally appropriate, research- and evidence-based assessment and instructional practices to promote all students' development of grade-level vocabulary knowledge and skills.

22. The teacher's practices in this scenario best demonstrate which of the following key principles of effective vocabulary instruction for prekindergarten children as described in the Texas Prekindergarten Guidelines?

  1. using young children's home language as a foundation for building their vocabulary
  2. creating ways for young children to interact with and use new vocabulary in meaningful contexts
  3. providing young children with feedback to correct their use of new vocabulary and to clarify meanings, if necessary
  4. encouraging young children to respond to questions about new vocabulary and to relate new vocabulary to their prior experiences
Enter to expand or collapse answer.Answer expanded
Option B is correct because the teacher in the scenario is using a read-aloud of an informational text and hands-on experiments related to the text to support children's vocabulary and concept development and purposeful language use. This approach supports their understanding of new words and concepts, as well as deepens their understanding of words and concepts they may already know. According to convergent research in vocabulary instruction, children learn new vocabulary more effectively when it is presented in meaningful contexts and the children have frequent opportunities to interact with and use the new vocabulary words purposefully. Several meaningful contexts occurred in the scenario, including the read-aloud, the classroom experiments, and potentially the extension of the window-garden experiments to children's homes. Option A is incorrect because the scenario focused on teaching the children new academic vocabulary words with high utility in STEM fields. It did not focus on building on children's home language. Option C is incorrect because the focus of the scenario was on creating contexts for the children to use new academic vocabulary in the school and home. It did not focus on corrective-feedback strategies. Option D is incorrect because the focus of the scenario was not on helping the children respond to questions about the vocabulary or relating the vocabulary to their prior experiences, but rather on creating new experiences in which the students could use the new vocabulary purposefully.

23. The extension of the activity best demonstrates the teacher's understanding of which of the following key factors affecting vocabulary development in prekindergarten children?

  1. the benefits of encouraging families to have literacy play a prominent role in their household
  2. the interrelationships between vocabulary development and concept learning
  3. the role of families in supporting and reinforcing young children's vocabulary development
  4. the importance of making abstract vocabulary words concrete for young children
Enter to expand or collapse answer.Answer expanded
Option C is correct because young children are more likely to use new words and incorporate them into their everyday vocabulary, if the words are used and reinforced in the home as well as in school. By sending home materials and instructions for creating a window garden, the teacher is helping to create a context in which use of the new vocabulary can be extended to the home. Option A is incorrect because the focus of the family extension activity was to recreate the hands-on learning activity in the home. It did not explicitly involve promoting literacy in the home. Option B is incorrect because the extension activity focuses on reinforcing the new vocabulary in the home rather than on developing new concepts related to the vocabulary. This step already took place during the original activities at school. Option D is incorrect because the teacher had already made the abstract vocabulary words concrete through the read-aloud and planting activities at school. The extension activity is meant to be a reinforcement activity.

Use the information below to answer the two questions that follow.

A kindergarten teacher wants to implement a focused-rereading protocol during read-alouds to introduce students to various strategies for understanding a complex text. The teacher completes a first reading of the text Memoirs of a Goldfish by Devin Scillian. During this reading, the teacher focuses on students' general understanding of the story. The story, which is written as a progressive diary, relates the plight and reactions of a goldfish as its fishbowl becomes increasingly crowded with various objects and creatures.

Competency 010—(Comprehension Development): Understand concepts, principles, and best practices related to the development of reading comprehension, and demonstrate knowledge of developmentally appropriate, research- and evidence-based assessment and instructional practices to promote all students' development of reading comprehension strategies in order to gain, clarify, and deepen understanding of appropriately complex texts.

24. During the second reading of the text, the teacher would like to focus students' attention on analyzing the author's craft. How can the teacher best achieve this goal?

  1. by using the illustrations and text to discuss the feelings and reactions of the main character
  2. by prompting students to retell the story in the order that the events occur (e.g., First, blank line. Next, blank line.)
  3. by rereading very expressively the parts of the text that are written in quotations
  4. by discussing how the text is constructed (e.g., who narrates the text; the use of dialogue, word choice, and diary structure)
Enter to expand or collapse answer.Answer expanded
Option D is correct because analyzing an author's craft includes analyzing text structure, word choice, narration, and dialogue. During the discussion, the teacher can use text-dependent questions aimed at author's craft to support students in understanding the text on a deeper level. For example, a unique feature of the text described in the scenario is that it uses a diary structure as a way to tell the story in the first person. A discussion of why the author chose to structure the text this way or chose to use certain words can help kindergarten students better understand a character's feelings, motivations, and reactions; while discussing the use of dialogue can help them better understand both characters and key story events. Option A is incorrect because discussing the feelings and reactions of the main character focuses students more on story plot and character development than on the choices the author made in constructing the text. Option B is incorrect because retelling the story in appropriate sequence focuses students on plot rather than on the author's craft. Option C is incorrect because rereading the dialogues expressively supports students' development of prosody, but it does not help the students understand how the author uses dialogue to move the story along or to convey information.

25. After the third reading of the text, the teacher has students discuss whether the goldfish was happier when it was once again alone in a fishbowl or when it rejoined the other creatures in a large fish tank. The students must work with a partner to locate support from the text and illustrations for their claim. Conducting this type of collaborative conversation as part of a focused-rereading protocol benefits students' understanding of a complex text primarily by:

  1. encouraging students to slow down and pay closer attention to the text.
  2. encouraging students to co-construct meaning using evidence from the text.
  3. promoting the use of questioning as a way to improve understanding of the text.
  4. promoting the use of visualization and mental imagery to clarify the text's meaning.
Enter to expand or collapse answer.Answer expanded
Option B is correct because when students interact with their peers about a text, they can help facilitate one another's comprehension of the text. Understanding of the text is co-constructed as the students ask each other questions or challenge each other with an argument or claim. Having them locate support from the text and illustrations deepens their analysis and raises their accountability with respect to the claims they make. Option A is incorrect because in this scenario, the students listened to the text during multiple, focused teacher read-alouds. It is not up to the students to "slow down," since the teacher is doing the reading. Options C and D are incorrect because the scenario does not allude to the students generating questions or using visualization or mental imagery to better understand the text. The teacher promotes their deeper understanding of the text by asking them to discuss the text with a partner, develop a claim about the main character, and look for evidence in the text and illustrations to support their claim.

Additional Selected-Response Questions

This section includes additional sample selected-response questions for you to review in preparation for the exam. The correct answer is provided for each question below.

Domain I—Reading Pedagogy

Competency 001—(Foundations of the Science of Teaching Reading): Understand foundational concepts, principles, and best practices related to the science of teaching reading.

26. A first-grade teacher provides reading instruction that is systematic and explicit and emphasizes both foundational reading skills and various dimensions of comprehension. According to research in preventing reading difficulties, which of the following additional actions would be most important for the teacher to take to ensure that reading instruction addresses all students' reading needs?

  1. engaging students in frequent oral reading activities to develop reading fluency
  2. exposing students to a variety of genres to enhance reading comprehension
  3. providing students with daily opportunities to work independently on written exercises that emphasize key grade-level reading and spelling skills
  4. assessing students' reading development regularly to implement timely and effective instructional responses when a delay is apparent
Enter to expand or collapse answer.Answer expanded
Option D is correct.

27. At the beginning of the school year, a kindergarten teacher determines that some students have had limited prior formal and informal literacy experiences. To accelerate these students' reading development, which of the following strategies would be most appropriate for the teacher to emphasize?

  1. conducting reading interest surveys with the students to match individual students with texts that they are most likely to enjoy reading during independent reading in class and for at-home reading-skills practice
  2. engaging the students in interactive read-alouds using predictable big books that feature rhyming words to promote their development of concepts of print, phonological awareness, and knowledge of text structure
  3. providing the students with intensive instruction in phonics skills, syllable patterns, and morphology using leveled texts to ensure their development of accurate, automatic decoding skills and reading fluency
  4. having the students help put up a classroom word wall of the 100 words that appear most frequently in children's texts to encourage them to begin learning to read the words automatically by sight
Enter to expand or collapse answer.Answer expanded
Option B is correct.

Competency 002—(Foundations of Reading Assessment): Understand foundational concepts, principles, and best practices related to reading assessment.

28. A second-grade teacher periodically conducts reading interest surveys with individual students. The teacher could best use the results of these assessments for which of the following instructional purposes?

  1. determining the level of parent/guardian involvement in students' literacy development
  2. assisting students in selecting books for independent reading time
  3. selecting texts at students' instructional reading level for small-group reading lessons
  4. identifying appropriate reading activities for students to complete at home
Enter to expand or collapse answer.Answer expanded
Option B is correct.

Domain II—Reading Development: Foundational Skills

Competency 004—(Phonological and Phonemic Awareness): Understand concepts, principles, and best practices related to the development of phonological and phonemic awareness, and demonstrate knowledge of developmentally appropriate, research- and evidence-based assessment and instructional practices to promote all students' development of grade-level phonological and phonemic awareness skills.

29. A prekindergarten teacher frequently engages children in circle time activities such as the activities described below.

  1. The teacher leads children in clapping the syllables of each classmate's name.
  2. The teacher helps children count how many syllables they hear in their classmates' names.
  3. The teacher has children with the same number of syllables in their names stand up and clap their classmates' names as a group.

According to convergent research, activities such as these are most effective in helping young children:

  1. recognize that their name has sounds that can be represented by the letters of the alphabet.
  2. discriminate individual speech sounds in names to help facilitate their learning how to write their own name.
  3. build phonological sensitivity by attending to the phonological structure of meaningful words such as names.
  4. connect the letters in their name to the sounds each of the letters makes.
Enter to expand or collapse answer.Answer expanded
Option C is correct.

Competency 005—(Print Concepts and Alphabet Knowledge): Understand concepts, principles, and best practices related to the development of print concepts and alphabet knowledge, including understanding of the alphabetic principle, and demonstrate knowledge of developmentally appropriate, research- and evidence-based assessment and instructional practices to promote all students' development of grade-level print concepts and alphabet knowledge and their understanding of the alphabetic principle.

30. A kindergarten teacher meets with individual students and asks them to point to the words in the text of a familiar nursery rhyme as the teacher and student read the nursery rhyme aloud together. Some students demonstrate understanding of the directionality of print by sweeping their finger as they "read," but they are not able to accurately point to the individual words. Other students who readily associate letters with sounds use this understanding to guide their finger as they point to a word that starts with the sound they hear at the beginning of the spoken word. The second group of students clearly try to match their speech to the print as they say the words. The teacher can best use the results of this informal assessment to determine which students are able to:

  1. apply key concepts related to the alphabetic principle.
  2. identify accurately the sounds of each letter of the alphabet.
  3. perform phonological awareness tasks at the syllable level.
  4. demonstrate advanced knowledge of key concepts of print.
Enter to expand or collapse answer.Answer expanded
Option A is correct.

31. A third-grade teacher reviews data on the literacy skills of several beginning-level English learners who did not attend school prior to moving to the United States. The teacher wants to plan appropriate small-group reading instruction for the students. Which of the following types of text would best meet the reading development needs of English learners who are at an emergent stage of English language development and at the pre-alphabetic phase of word reading?

  1. decodable stories
  2. informational text
  3. predictable text
  4. age-appropriate stories
Enter to expand or collapse answer.Answer expanded
Option C is correct.

Competency 006—(Phonics and Other Word Identification Skills): Understand concepts, principles, and best practices related to the development of phonics and other word identification skills, including related spelling skills, and demonstrate knowledge of developmentally appropriate, research- and evidence-based assessment and instructional practices to promote all students' development of grade-level phonics and other word identification skills and related spelling skills.

32. A first-grade teacher is working with a small group of students that includes English learners and speakers of various dialects of English. As part of a series of lessons on the inflectional ending -ed, the teacher helps the students sort a list of inflected verbs according to their final sound or syllable. The teacher selected the verbs from a text the students are currently reading. The students' completed word-sort chart is shown below.

Sorting Verbs with the -ed Inflection
/t/ /d/ left bracket schwa d right bracket
kicked tagged rested
tossed yelled landed
poked grabbed dented
missed planned planted
gasped jammed ended

The teacher's strategy of having the students sort and pronounce inflected verbs is likely to benefit the English learners and speakers of various dialects primarily in which of the following ways?

  1. by providing the students with multiple opportunities to encode the verbs to promote fluent writing
  2. by helping the students learn to perceive and produce inflections that they may not use in their everyday speech
  3. by providing the students with extra practice applying phonics patterns in their reading and writing
  4. by helping the students make a connection between the spelling and meaning of a variety of common English verbs
Enter to expand or collapse answer.Answer expanded
Option B is correct.

Competency 007—(Syllabication and Morphemic Analysis Skills): Understand concepts, principles, and best practices related to the development of syllabication and morphemic analysis skills, including related spelling skills, and demonstrate knowledge of developmentally appropriate, research- and evidence-based assessment and instructional practices to promote all students' development of grade-level syllabication and morphemic analysis skills and related spelling skills.

33. A second-grade teacher divides the class into pairs and presents each pair of students with a unique set of word cards. The teacher models how to match two word cards to build a new word (e.g., mail plus box equals mailbox, some plus thing equals something). The teacher then challenges the students to work with their partners to build as many new words as they can using their own set of word cards. The students are assigned to make a list of their new words and draft a sentence for three of the words. Afterward, each pair of students reads aloud their lists of words and sentences to the class. This activity supports students' reading development primarily by promoting their ability to:

  1. determine the meaning of words containing common roots and affixes.
  2. read aloud texts with appropriate accuracy, rate, and prosody.
  3. self-correct errors when reading multisyllabic words in connected text.
  4. decode compound words quickly and accurately while reading.
Enter to expand or collapse answer.Answer expanded
Option D is correct.

Competency 008—(Reading Fluency): Understand concepts, principles, and best practices related to the development of reading fluency, and demonstrate knowledge of developmentally appropriate, research- and evidence-based assessment and instructional practices to promote all students' development of grade-level reading fluency.

34. A second-grade teacher is working with students to develop their automaticity in recognizing high-frequency words. Several English learners often misread or omit high-frequency prepositions (e.g., in, on, of, by) when reading connected text. Which of the following strategies for differentiating instruction for the English learners would best scaffold their learning in order to promote their accuracy and automaticity in reading high-frequency, grade-level function words?

  1. constructing word grids of the target words so that the students can practice reading the words in isolation before reading a text that includes them
  2. having the students practice reading the target words in meaningful phrases that are illustrated to reinforce understanding
  3. pointing out the parts of each target word that are and are not decodable and providing additional practice with the non-decodable elements
  4. engaging the students in a timed collaborative match game with a partner in which they try to improve their collective rate matching pairs of the target words
Enter to expand or collapse answer.Answer expanded
Option B is correct.

Domain III—Reading Development: Comprehension

Competency 010—(Comprehension Development): Understand concepts, principles, and best practices related to the development of reading comprehension, and demonstrate knowledge of developmentally appropriate, research- and evidence-based assessment and instructional practices to promote all students' development of reading comprehension strategies in order to gain, clarify, and deepen understanding of appropriately complex texts.

35. A kindergarten teacher would like to determine if students can make inferences about characters and/or events in stories they hear or read. Which of the following assessment approaches is most likely to require students to make inferences?

  1. asking students to identify an important event in the story and to provide details about a character's actions during that event
  2. asking students to answer general understanding questions about the story based on evidence from rereading the text and/or reviewing the illustrations
  3. asking students to use evidence from the text and illustrations to explain how a character feels and why the character feels that way
  4. asking students to retell the events of the story, first by using an open-ended question, and then by following up with a focus question about event sequences
Enter to expand or collapse answer.Answer expanded
Option C is correct.

36. A third-grade teacher would like to model how to apply metacognitive strategies while reading in order to improve students' reading comprehension. Which of the following strategies would be most appropriate to model?

  1. making predictions and self-questioning while reading
  2. inferring the meaning of new vocabulary words while reading
  3. identifying the main idea and key details while reading
  4. decoding with automaticity to maintain fluency while reading
Enter to expand or collapse answer.Answer expanded
Option A is correct.

Competency 011—(Comprehension of Literary Texts): Understand concepts, principles, and best practices related to the comprehension of and critical thinking about literary texts, and demonstrate knowledge of developmentally appropriate, research- and evidence-based assessment and instructional practices to promote all students' development of grade-level comprehension and analysis skills for literary texts.

37. A prekindergarten teacher frequently reads aloud high-quality literature to develop children's familiarity with literary texts. Which of the following accompanying activities would best promote the children's understanding of basic story structure?

  1. having the children use props to act out a scene from the story
  2. pre-teaching unfamiliar vocabulary from the story to support comprehension
  3. stopping at appropriate places in the story to make predictions
  4. giving the children picture cards of events in the story to retell in sequence
Enter to expand or collapse answer.Answer expanded
Option D is correct.

Competency 012—(Comprehension of Informational Texts): Understand concepts, principles, and best practices related to the comprehension of and critical thinking about informational texts, and demonstrate knowledge of developmentally appropriate, research- and evidence-based assessment and instructional practices to promote all students' development of grade-level comprehension and analysis skills for informational texts.

38. To promote students' ability to engage in academic conversations about informational texts, a second-grade teacher creates the following anchor chart with students. The teacher posts it in the classroom and supports students in using it during discussions.

For example, ...

The author said ...

According to the author, ...

From the reading I know that ...

On page blank, the author says ...

For instance, ...

Because ...

This list of phrases is best designed to scaffold students' ability to apply which of the following skills during text-based discussions about informational texts?

  1. using textual evidence to support claims
  2. applying metacognitive strategies
  3. distinguishing facts from opinions
  4. analyzing an author's craft
Enter to expand or collapse answer.Answer expanded
Option A is correct.

39. A first-grade teacher often uses a simple timeline graphic organizer to scaffold information for students when they are reading social studies texts about historical events or the lives of important people. The teacher's practice is most likely to promote students' development of which of the following disciplinary-literacy skills?

  1. recognizing text structures commonly used in social studies
  2. identifying discipline-specific text features commonly used in social studies
  3. evaluating sources and using evidence in social studies texts
  4. distinguishing discipline-specific meanings of words used in social studies and other content areas
Enter to expand or collapse answer.Answer expanded
Option A is correct.

Clustered Questions

Use the information below to answer the two questions that follow.

A first-grade classroom includes several students who are English learners. The teacher is preparing text-dependent prompts to ask students during a read-aloud of the big book The Napping House by Audrey Wood. The first set of prompts focuses on establishing the setting of the story. The text describes a house where everyone is sleeping. The illustration shows a granny sleeping in a bed, a cat sleeping on a chair, and a dog sleeping on the floor. It is raining outside the window.

Competency 003—(Oral Language Foundations of Reading Development): Understand foundational concepts, principles, and best practices related to the development of oral language, including second-language acquisition, and demonstrate knowledge of developmentally appropriate, research- and evidence-based assessment and instructional practices to promote all students' development of grade-level oral language skills.

40. The teacher wants to align prompts about the setting of the story with the English learners' oral proficiency levels. Which of the following prompts would best align with a student who is at the beginning level of oral language proficiency in English?

  1. Describe the setting of the story.
  2. Who is sleeping in the room?
  3. What is happening outside the house?
  4. Point to where everyone is sleeping.
Enter to expand or collapse answer.Answer expanded
Option D is correct.

41. As the story progresses, more characters enter the bedroom and pile on the snoring granny to take a nap. The teacher asks students to describe what is occurring on each page. In response to the text and illustrations, one English learner says, "Mouse sleep on cat. Cat sleep on dog." The student's grammar is most typical of an English learner at which of the following levels of English language proficiency?

  1. beginning
  2. intermediate
  3. advanced
  4. advanced high
Enter to expand or collapse answer.Answer expanded
Option B is correct.

Use the information below to answer the two questions that follow.

A first-grade teacher conducts a series of phonics lessons with a small group of students. The teacher supports instruction by providing the students with oral reading practice using decodable texts that feature the phonics skill being taught. After providing instruction and guided practice in the target phonics skill, the teacher administers a 10-word oral reading assessment to individual students in the group. The assessment includes target words from the decodable texts the students read as well as new words that are unfamiliar to the students but that require them to use the same phonics skill targeted in the lessons. The data for one student are shown below.

Target Word from Text Student Reads Target New Word Student Reads
lift lift left let
quilt quit quest quet
twist twit twink twik
flap flap fled fled

Competency 002—(Foundations of Reading Assessment): Understand foundational concepts, principles, and best practices related to reading assessment.

42. The teacher uses the assessment in this scenario for which of the following assessment purposes?

  1. monitoring students' progress toward mastery of a reading skill
  2. establishing students' baseline performance with respect to a reading skill to be taught
  3. determining whether students have achieved grade-level reading skills
  4. comparing students' growth with respect to a reading skill before and after instruction
Enter to expand or collapse answer.Answer expanded
Option A is correct.

Competency 006—(Phonics and Other Word Identification Skills): Understand concepts, principles, and best practices related to the development of phonics and other word identification skills, including related spelling skills, and demonstrate knowledge of developmentally appropriate, research- and evidence-based assessment and instructional practices to promote all students' development of grade-level phonics and other word identification skills and related spelling skills.

43. Given the student's performance on the assessment, which of the following actions would be most appropriate for the teacher to take next?

  1. implementing reinforcement activities with the student focused on common consonant digraphs
  2. engaging the student in phonemic awareness activities focused on final consonant blends
  3. sending home the words the student missed to practice for reading homework
  4. transitioning the student to reading long-vowel-pattern words
Enter to expand or collapse answer.Answer expanded
Option B is correct.

Use the information below to answer the two questions that follow.

A second-grade teacher analyzes the summary results of a student's oral reading fluency measure, shown below.

Book level:  Early second grade
Accuracy rate:  92%

Error rate:  1:12 (average of 1 error for every 12 words read)

Self-correction rate:  1:6 (average of 1 self-correction for every 6 errors)

After reviewing the summary results, the teacher has a conference with the student. A transcript of part of their conference is shown below.

Teacher:  I noticed that you used some very good strategies. When you read the word of instead of off, you could tell it didn't sound right in the sentence, and you fixed it right away. Of and off look a little alike, but they're not the same. I also noticed that you were thinking about what is happening in the story, and you used that to help decide whether a word you read makes sense. Let's look at this word right here (pointing to the word rolls in the text). You read goes. Let's check the first letter. What is it?

Student:  It's r. Oh! It says rolls!

Teacher:  Yes! When you read goes, it made sense, but it didn't sound right. Rolls makes sense, and it also sounds right.

Competency 002—(Foundations of Reading Assessment): Understand foundational concepts, principles, and best practices related to reading assessment.

44. The student-teacher conference described in this scenario best reflects the teacher's awareness of the importance of:

  1. using the results of assessments to support students' choices for independent reading through discussions about their interests.
  2. providing feedback to students in ways that encourage, support, and motivate them in their reading development.
  3. determining students' reading levels in order to support the selection of instructional resources.
  4. differentiating reading assessments for individual students in order to assess all students accurately and objectively.
Enter to expand or collapse answer.Answer expanded
Option B is correct.

Competency 006—(Phonics and Other Word Identification Skills): Understand concepts, principles, and best practices related to the development of phonics and other word identification skills, including related spelling skills, and demonstrate knowledge of developmentally appropriate, research- and evidence-based assessment and instructional practices to promote all students' development of grade-level phonics and other word identification skills and related spelling skills.

45. Given the information provided, which of the following instructional plans would likely be most effective in improving this student's ability to self-monitor and self-correct while reading?

  1. grouping the student with other students who are struggling with self-correcting and then explicitly reteaching skills involved in the process
  2. providing peer-to-peer support by grouping the student for reading instruction with students who consistently self-correct
  3. grouping the student with other students who are struggling with self-correcting and then having them read and reread lower-level texts
  4. providing more opportunities for self-correction by grouping the student with students who are reading at a higher level
Enter to expand or collapse answer.Answer expanded
Option A is correct.

Use the information below to answer the two questions that follow.

Students in a second-grade class have been learning about synonyms. As part of an introduction to the term antonym, the teacher reads aloud the book If You Were an Antonym by Nancy Loewen. In this book, the author explains what an antonym is and provides many examples of different types of antonyms. After discussing the book with students, the teacher models how to create a semantic map of a word's synonyms and antonyms using a simple two-column format. The teacher writes the word "cold" at the top of the chart and prompts students to help generate words for each column. The teacher encourages the students to think of words from the book the teacher just read aloud as well as from other books the students have recently read in class. An excerpt from their completed chart is shown below.

Target Word: Cold
Synonyms Antonyms
chilly hot
freezing burning
frosty fiery

Afterward, the teacher supports students in generating similar semantic charts for the vocabulary words that they are studying as part of their current weekly word study.

Competency 009—(Vocabulary Development): Understand concepts, principles, and best practices related to vocabulary development, and demonstrate knowledge of developmentally appropriate, research- and evidence-based assessment and instructional practices to promote all students' development of grade-level vocabulary knowledge and skills.

46. This type of activity benefits students' vocabulary development most directly by:

  1. enhancing their ability to engage in self-sustained reading of texts that contain challenging words.
  2. promoting their understanding of the relationships between words.
  3. facilitating their ability to distinguish between a word's denotative and connotative meanings.
  4. helping them analyze an author's craft with respect to word choice.
Enter to expand or collapse answer.Answer expanded
Option B is correct.

47. The teacher would like to reinforce this lesson while also promoting the students' knowledge of independent word-learning strategies. The teacher could best address both goals by showing the students how to:

  1. apply contextual analysis to determine the meaning of one of the synonyms or antonyms in connected text.
  2. apply structural/morphemic analysis to one of the synonyms or antonyms of a target word.
  3. use an online etymological dictionary to research the origin of one of the synonyms or antonyms.
  4. use print and/or digital resources to search for more synonyms and antonyms of a target word.
Enter to expand or collapse answer.Answer expanded
Option D is correct.

Use the information below to answer the three questions that follow.

A second-grade class includes several English learners whose home language is Spanish. For a series of whole-class lessons focused on promoting students' ability to analyze story relationships, the teacher selects the text Carlos and the Squash Plant (Carlos y la planta de calabaza) by Jan Romero Stevens, which has a side-by-side translation of the story in English and Spanish. In the story, the main character ignores his mother's warning that a squash plant will grow from his ears if he does not wash them, so a squash plant does indeed begin growing from his ear. The teacher invites a parent/guardian to read the Spanish version to the class.

After the story is read-aloud to the students, the teacher conducts an analysis of the story with students with regard to key story elements (e.g., setting, characters, plot, message). The teacher then introduces students to a Somebody-Wanted-Because-But-So-Then chart to facilitate their ability to develop written summaries of the events in a story in a way that conveys story relationships. The prompts in the chart encourage students to consider the plot of a narrative text with regard to the main character's goals, motivations, and related actions.

Students work with a partner to complete the right column of the chart and then develop a written summary of the text. The teacher's anchor model is shown below.

Summarizing Prompt Model Summary Statement
Somebody wanted... Carlos wanted to remove the squash plant that was growing from his ear.
Because... Because he did not want his mother to find out that he had disobeyed her about taking a bath.
But... But no matter how hard he pulled, he could not remove the squash plant.
So... So he decided to wear a hat. When the squash plant grew larger, he put on a larger hat.
Then... Then, when the wind blew the hat away, he finally decided to scrub his ears. This made the squash plant disappear.

Competency 010—(Comprehension Development): Understand concepts, principles, and best practices related to the development of reading comprehension, and demonstrate knowledge of developmentally appropriate, research- and evidence-based assessment and instructional practices to promote all students' development of reading comprehension strategies in order to gain, clarify, and deepen understanding of appropriately complex texts.

48. The read-aloud portion of the lesson best demonstrates the teacher's understanding of the importance of selecting texts for reading instruction that:

  1. contribute to students' development of academic vocabulary.
  2. provide opportunities for making text-to-text and text-to-world connections.
  3. reflect the diversity of the classroom and school communities.
  4. ensure students' exposure to multiple genres of literary texts.
Enter to expand or collapse answer.Answer expanded
Option C is correct.

Competency 011—(Comprehension of Literary Texts): Understand concepts, principles, and best practices related to the comprehension of and critical thinking about literary texts, and demonstrate knowledge of developmentally appropriate, research- and evidence-based assessment and instructional practices to promote all students' development of grade-level comprehension and analysis skills for literary texts.

49. The teacher observes that some students are having difficulty completing the right side of the blank Somebody-Wanted-Because-But-So-Then chart. Which of the following scaffolding strategies would best help the students use the chart to develop summaries that convey story relationships?

  1. having the students retell the story using sequence words (e.g., first, next, then, after that, finally) before writing a summary
  2. adding a text-dependent question next to each of the summarizing prompts (e.g., "Because..." / "Why didn't Carlos want to ask his mother for help?")
  3. revisiting the students' general understanding of the story using text-dependent questions at the literal level
  4. providing additional graphic organizers, such as a concept map or organizers for cause/effect, problem/solution, and comparison/contrast
Enter to expand or collapse answer.Answer expanded
Option B is correct.

50. The teacher differentiates the written-summary portion of the lesson for the English learners by having them practice orally retelling the story before they develop their written summaries. The teacher listens to their retelling and provides them with feedback, coaching, and/or additional instruction, as needed. The transcript of one intermediate English learner's oral retelling of the story is shown below.

Student:  Mom say, "Take a bath Carlos." Carlos don't take a bath. The next day a calabacita is coming out his ear. He feel scared his mom be mad. He put a hat over the calabacita. Then calabacita plant get more big, and he try to take it out. Next, he got a more bigger hat. Then, the wind blow away his hat! Finally, he take a bath, and he washing his ears, and he scrubbing hard. Then the calabacita go away!

Given this retelling, the teacher could best improve the student's ability to summarize the story in a way that conveys story relationships by providing explicit instruction in which of the following skills?

  1. applying comparative suffixes to convey size differences and differences in the intensity of actions
  2. changing verbs from the story into past tense to convey that the events already took place
  3. incorporating sequence words and phrases to support retelling the story in the correct sequence
  4. using causal conjunctions to connect the various events in the story in a logical manner
Enter to expand or collapse answer.Answer expanded
Option D is correct.

Return to Navigation